Finding a Matrix whose kernel is spanned by 2 vectors

Click For Summary
To find a matrix whose kernel is spanned by the vectors u=(1,3,2) and v=(-2,0,4), the discussion suggests identifying a vector w that is orthogonal to both u and v. The user initially attempted to create a matrix using u and v but encountered issues with the row reduction process, leading to incorrect conclusions about the kernel. The key idea is to construct a matrix where all rows are the orthogonal vector w, ensuring that both u and v lie in the kernel. The correct approach involves ensuring that the resulting matrix satisfies the condition that M*u and M*v equal zero, confirming that both vectors are indeed in the kernel.
DanceLink
Messages
5
Reaction score
0

Homework Statement


Find a matrix whose kernel is spanned by the two vectors u=(1,3,2) and v=(-2,0,4).


Homework Equations





The Attempt at a Solution


Tried setting vectors as a matrix and rref'ing it, but didn't know where I was getting at, also tried using an augmented identity matrix with both vectors then realized I didn't know what I was doing.
 
Physics news on Phys.org
How about finding a vector w that's orthogonal to both u and v and making a matrix where all of the rows are w?
 
  • Like
Likes Sushil Khatri
k I think I got it...
Given:
u=[1,3,2]
v=[-2,0,4]

I put in the vector M(1)=[1,1,1]
M:=<u|v|M(1)>
and I get...
[1,-2,1
3,0,1
2,4,1]

I apply rref, and I get an identity matrix. Meaning that the above matrix is my answer, right?
 
That doesn't work. M*u is (-3,5,14) isn't it? If u is in the kernel M*u is supposed to (0,0,0). I don't think you heard me. Find a vector w so that u.w=0 and v.w=0. Wouldn't it work if you make a matrix with all of the rows w?
 
Question: A clock's minute hand has length 4 and its hour hand has length 3. What is the distance between the tips at the moment when it is increasing most rapidly?(Putnam Exam Question) Answer: Making assumption that both the hands moves at constant angular velocities, the answer is ## \sqrt{7} .## But don't you think this assumption is somewhat doubtful and wrong?

Similar threads

  • · Replies 14 ·
Replies
14
Views
1K
  • · Replies 1 ·
Replies
1
Views
2K
  • · Replies 1 ·
Replies
1
Views
1K
Replies
9
Views
15K
  • · Replies 2 ·
Replies
2
Views
2K
Replies
2
Views
2K
  • · Replies 2 ·
Replies
2
Views
2K
  • · Replies 8 ·
Replies
8
Views
2K
  • · Replies 13 ·
Replies
13
Views
2K
  • · Replies 9 ·
Replies
9
Views
2K